$mathbb{E}X^2 = mathbb{E}Y^2$ and $mathbb{E}(Y|mathcal{A}) = X$ P-a.s. $Rightarrow X=Y$ P.-a.s.












1












$begingroup$


I have a question and hope you can help me. The problem is about stochastic variables $X,Y$, which are square integrable, independent and identically distributed on $(Omega, mathcal{S}, P)$. Furthermore $mathcal{A} subsetmathcal{S}$ is a sub-sigma-algebra.
Now I have to show three statements:



a) $mathbb{V}(X) = mathbb{E}(X|mathcal{A}) + mathbb{E}[X-mathbb{E}(X|mathcal{A})]^2 Rightarrow mathbb{V}(mathbb{E}(X|mathcal{A})) leq mathbb{V}(X)$



b) $mathbb{E}X^2 = mathbb{E}Y^2$ and $mathbb{E}(Y|mathcal{A}) = X$ P-a.s. $Rightarrow X=Y$ P.-a.s.



c) $mathbb{E}(X|Y) = Y$ P-a.s. and $mathbb{E}(Y|X) = X$ P-a.s. $Rightarrow X=Y$ P-a.s.



I already managed to proof a) by using variance-decomposition-theorem and a little bit of calculation.



Furthermore I think I have an idea how to solve c):



$mathbb{E}[XY|X] = X mathbb{E}[Y|X] = X X = X^2$ P.-almost sure, where the second equal-sign uses the assumption. Similarly there one can conclude $mathbb{E}[XY|Y] = Y^2$ P-a.s.



Applying $mathbb{E}$ on both sides gives me:
$mathbb{E}[X^2]=mathbb{E}[mathbb{E}[XY|X]] = mathbb{E}[XY] = mathbb{E}[mathbb{E}[XY|Y]] = mathbb{E}[Y^2]$ P-a.s. Now, by using this result, I can conclude $mathbb{E}[(X-Y)^2] = 0$ to get the required result.



Sadly I don't have a clue how to prove b) to finish this task.



I would be very glad if anyone could help me solving b) too!



Thanks in advance for your help!










share|cite|improve this question











$endgroup$












  • $begingroup$
    Implication c) holds for integrable random variables X and Y not necessarily square integrable, as shown in answers to several questions on this site.
    $endgroup$
    – Did
    Dec 17 '18 at 22:19










  • $begingroup$
    @Did I believe c) it to be true in general, but I failed to provide a proof. Do you have any of this links or a proof-sketch?
    $endgroup$
    – Will M.
    Dec 17 '18 at 22:37










  • $begingroup$
    I thought, that my proof for c) (given above) is valid. Is there anything wrong with it? The only step I need square integrability is (at least I think so) $mathbb{E}[XY|X] = X mathbb{E}[Y|X]$. With Hölder's inequality I can conclude, that if X,Y are square-integrable then XY is integrable: $int XY dP leq int |XY| dp stackrel{Hölder}{=} left( int |X|^2 dP right)^{frac{1}{2}} left( int |Y|^2 dP right)^{frac{1}{2}} < infty$. Or am I wrong?
    $endgroup$
    – TNicky
    Dec 17 '18 at 22:42












  • $begingroup$
    It is true that if $X$ and $Y$ are square integrable, then $XY$ is integrable.
    $endgroup$
    – Will M.
    Dec 18 '18 at 1:36










  • $begingroup$
    @WillM. math.stackexchange.com/a/1520480/6179
    $endgroup$
    – Did
    Dec 18 '18 at 7:20
















1












$begingroup$


I have a question and hope you can help me. The problem is about stochastic variables $X,Y$, which are square integrable, independent and identically distributed on $(Omega, mathcal{S}, P)$. Furthermore $mathcal{A} subsetmathcal{S}$ is a sub-sigma-algebra.
Now I have to show three statements:



a) $mathbb{V}(X) = mathbb{E}(X|mathcal{A}) + mathbb{E}[X-mathbb{E}(X|mathcal{A})]^2 Rightarrow mathbb{V}(mathbb{E}(X|mathcal{A})) leq mathbb{V}(X)$



b) $mathbb{E}X^2 = mathbb{E}Y^2$ and $mathbb{E}(Y|mathcal{A}) = X$ P-a.s. $Rightarrow X=Y$ P.-a.s.



c) $mathbb{E}(X|Y) = Y$ P-a.s. and $mathbb{E}(Y|X) = X$ P-a.s. $Rightarrow X=Y$ P-a.s.



I already managed to proof a) by using variance-decomposition-theorem and a little bit of calculation.



Furthermore I think I have an idea how to solve c):



$mathbb{E}[XY|X] = X mathbb{E}[Y|X] = X X = X^2$ P.-almost sure, where the second equal-sign uses the assumption. Similarly there one can conclude $mathbb{E}[XY|Y] = Y^2$ P-a.s.



Applying $mathbb{E}$ on both sides gives me:
$mathbb{E}[X^2]=mathbb{E}[mathbb{E}[XY|X]] = mathbb{E}[XY] = mathbb{E}[mathbb{E}[XY|Y]] = mathbb{E}[Y^2]$ P-a.s. Now, by using this result, I can conclude $mathbb{E}[(X-Y)^2] = 0$ to get the required result.



Sadly I don't have a clue how to prove b) to finish this task.



I would be very glad if anyone could help me solving b) too!



Thanks in advance for your help!










share|cite|improve this question











$endgroup$












  • $begingroup$
    Implication c) holds for integrable random variables X and Y not necessarily square integrable, as shown in answers to several questions on this site.
    $endgroup$
    – Did
    Dec 17 '18 at 22:19










  • $begingroup$
    @Did I believe c) it to be true in general, but I failed to provide a proof. Do you have any of this links or a proof-sketch?
    $endgroup$
    – Will M.
    Dec 17 '18 at 22:37










  • $begingroup$
    I thought, that my proof for c) (given above) is valid. Is there anything wrong with it? The only step I need square integrability is (at least I think so) $mathbb{E}[XY|X] = X mathbb{E}[Y|X]$. With Hölder's inequality I can conclude, that if X,Y are square-integrable then XY is integrable: $int XY dP leq int |XY| dp stackrel{Hölder}{=} left( int |X|^2 dP right)^{frac{1}{2}} left( int |Y|^2 dP right)^{frac{1}{2}} < infty$. Or am I wrong?
    $endgroup$
    – TNicky
    Dec 17 '18 at 22:42












  • $begingroup$
    It is true that if $X$ and $Y$ are square integrable, then $XY$ is integrable.
    $endgroup$
    – Will M.
    Dec 18 '18 at 1:36










  • $begingroup$
    @WillM. math.stackexchange.com/a/1520480/6179
    $endgroup$
    – Did
    Dec 18 '18 at 7:20














1












1








1





$begingroup$


I have a question and hope you can help me. The problem is about stochastic variables $X,Y$, which are square integrable, independent and identically distributed on $(Omega, mathcal{S}, P)$. Furthermore $mathcal{A} subsetmathcal{S}$ is a sub-sigma-algebra.
Now I have to show three statements:



a) $mathbb{V}(X) = mathbb{E}(X|mathcal{A}) + mathbb{E}[X-mathbb{E}(X|mathcal{A})]^2 Rightarrow mathbb{V}(mathbb{E}(X|mathcal{A})) leq mathbb{V}(X)$



b) $mathbb{E}X^2 = mathbb{E}Y^2$ and $mathbb{E}(Y|mathcal{A}) = X$ P-a.s. $Rightarrow X=Y$ P.-a.s.



c) $mathbb{E}(X|Y) = Y$ P-a.s. and $mathbb{E}(Y|X) = X$ P-a.s. $Rightarrow X=Y$ P-a.s.



I already managed to proof a) by using variance-decomposition-theorem and a little bit of calculation.



Furthermore I think I have an idea how to solve c):



$mathbb{E}[XY|X] = X mathbb{E}[Y|X] = X X = X^2$ P.-almost sure, where the second equal-sign uses the assumption. Similarly there one can conclude $mathbb{E}[XY|Y] = Y^2$ P-a.s.



Applying $mathbb{E}$ on both sides gives me:
$mathbb{E}[X^2]=mathbb{E}[mathbb{E}[XY|X]] = mathbb{E}[XY] = mathbb{E}[mathbb{E}[XY|Y]] = mathbb{E}[Y^2]$ P-a.s. Now, by using this result, I can conclude $mathbb{E}[(X-Y)^2] = 0$ to get the required result.



Sadly I don't have a clue how to prove b) to finish this task.



I would be very glad if anyone could help me solving b) too!



Thanks in advance for your help!










share|cite|improve this question











$endgroup$




I have a question and hope you can help me. The problem is about stochastic variables $X,Y$, which are square integrable, independent and identically distributed on $(Omega, mathcal{S}, P)$. Furthermore $mathcal{A} subsetmathcal{S}$ is a sub-sigma-algebra.
Now I have to show three statements:



a) $mathbb{V}(X) = mathbb{E}(X|mathcal{A}) + mathbb{E}[X-mathbb{E}(X|mathcal{A})]^2 Rightarrow mathbb{V}(mathbb{E}(X|mathcal{A})) leq mathbb{V}(X)$



b) $mathbb{E}X^2 = mathbb{E}Y^2$ and $mathbb{E}(Y|mathcal{A}) = X$ P-a.s. $Rightarrow X=Y$ P.-a.s.



c) $mathbb{E}(X|Y) = Y$ P-a.s. and $mathbb{E}(Y|X) = X$ P-a.s. $Rightarrow X=Y$ P-a.s.



I already managed to proof a) by using variance-decomposition-theorem and a little bit of calculation.



Furthermore I think I have an idea how to solve c):



$mathbb{E}[XY|X] = X mathbb{E}[Y|X] = X X = X^2$ P.-almost sure, where the second equal-sign uses the assumption. Similarly there one can conclude $mathbb{E}[XY|Y] = Y^2$ P-a.s.



Applying $mathbb{E}$ on both sides gives me:
$mathbb{E}[X^2]=mathbb{E}[mathbb{E}[XY|X]] = mathbb{E}[XY] = mathbb{E}[mathbb{E}[XY|Y]] = mathbb{E}[Y^2]$ P-a.s. Now, by using this result, I can conclude $mathbb{E}[(X-Y)^2] = 0$ to get the required result.



Sadly I don't have a clue how to prove b) to finish this task.



I would be very glad if anyone could help me solving b) too!



Thanks in advance for your help!







measure-theory conditional-expectation conditional-probability variance






share|cite|improve this question















share|cite|improve this question













share|cite|improve this question




share|cite|improve this question








edited Dec 17 '18 at 22:48







TNicky

















asked Dec 17 '18 at 20:57









TNickyTNicky

85




85












  • $begingroup$
    Implication c) holds for integrable random variables X and Y not necessarily square integrable, as shown in answers to several questions on this site.
    $endgroup$
    – Did
    Dec 17 '18 at 22:19










  • $begingroup$
    @Did I believe c) it to be true in general, but I failed to provide a proof. Do you have any of this links or a proof-sketch?
    $endgroup$
    – Will M.
    Dec 17 '18 at 22:37










  • $begingroup$
    I thought, that my proof for c) (given above) is valid. Is there anything wrong with it? The only step I need square integrability is (at least I think so) $mathbb{E}[XY|X] = X mathbb{E}[Y|X]$. With Hölder's inequality I can conclude, that if X,Y are square-integrable then XY is integrable: $int XY dP leq int |XY| dp stackrel{Hölder}{=} left( int |X|^2 dP right)^{frac{1}{2}} left( int |Y|^2 dP right)^{frac{1}{2}} < infty$. Or am I wrong?
    $endgroup$
    – TNicky
    Dec 17 '18 at 22:42












  • $begingroup$
    It is true that if $X$ and $Y$ are square integrable, then $XY$ is integrable.
    $endgroup$
    – Will M.
    Dec 18 '18 at 1:36










  • $begingroup$
    @WillM. math.stackexchange.com/a/1520480/6179
    $endgroup$
    – Did
    Dec 18 '18 at 7:20


















  • $begingroup$
    Implication c) holds for integrable random variables X and Y not necessarily square integrable, as shown in answers to several questions on this site.
    $endgroup$
    – Did
    Dec 17 '18 at 22:19










  • $begingroup$
    @Did I believe c) it to be true in general, but I failed to provide a proof. Do you have any of this links or a proof-sketch?
    $endgroup$
    – Will M.
    Dec 17 '18 at 22:37










  • $begingroup$
    I thought, that my proof for c) (given above) is valid. Is there anything wrong with it? The only step I need square integrability is (at least I think so) $mathbb{E}[XY|X] = X mathbb{E}[Y|X]$. With Hölder's inequality I can conclude, that if X,Y are square-integrable then XY is integrable: $int XY dP leq int |XY| dp stackrel{Hölder}{=} left( int |X|^2 dP right)^{frac{1}{2}} left( int |Y|^2 dP right)^{frac{1}{2}} < infty$. Or am I wrong?
    $endgroup$
    – TNicky
    Dec 17 '18 at 22:42












  • $begingroup$
    It is true that if $X$ and $Y$ are square integrable, then $XY$ is integrable.
    $endgroup$
    – Will M.
    Dec 18 '18 at 1:36










  • $begingroup$
    @WillM. math.stackexchange.com/a/1520480/6179
    $endgroup$
    – Did
    Dec 18 '18 at 7:20
















$begingroup$
Implication c) holds for integrable random variables X and Y not necessarily square integrable, as shown in answers to several questions on this site.
$endgroup$
– Did
Dec 17 '18 at 22:19




$begingroup$
Implication c) holds for integrable random variables X and Y not necessarily square integrable, as shown in answers to several questions on this site.
$endgroup$
– Did
Dec 17 '18 at 22:19












$begingroup$
@Did I believe c) it to be true in general, but I failed to provide a proof. Do you have any of this links or a proof-sketch?
$endgroup$
– Will M.
Dec 17 '18 at 22:37




$begingroup$
@Did I believe c) it to be true in general, but I failed to provide a proof. Do you have any of this links or a proof-sketch?
$endgroup$
– Will M.
Dec 17 '18 at 22:37












$begingroup$
I thought, that my proof for c) (given above) is valid. Is there anything wrong with it? The only step I need square integrability is (at least I think so) $mathbb{E}[XY|X] = X mathbb{E}[Y|X]$. With Hölder's inequality I can conclude, that if X,Y are square-integrable then XY is integrable: $int XY dP leq int |XY| dp stackrel{Hölder}{=} left( int |X|^2 dP right)^{frac{1}{2}} left( int |Y|^2 dP right)^{frac{1}{2}} < infty$. Or am I wrong?
$endgroup$
– TNicky
Dec 17 '18 at 22:42






$begingroup$
I thought, that my proof for c) (given above) is valid. Is there anything wrong with it? The only step I need square integrability is (at least I think so) $mathbb{E}[XY|X] = X mathbb{E}[Y|X]$. With Hölder's inequality I can conclude, that if X,Y are square-integrable then XY is integrable: $int XY dP leq int |XY| dp stackrel{Hölder}{=} left( int |X|^2 dP right)^{frac{1}{2}} left( int |Y|^2 dP right)^{frac{1}{2}} < infty$. Or am I wrong?
$endgroup$
– TNicky
Dec 17 '18 at 22:42














$begingroup$
It is true that if $X$ and $Y$ are square integrable, then $XY$ is integrable.
$endgroup$
– Will M.
Dec 18 '18 at 1:36




$begingroup$
It is true that if $X$ and $Y$ are square integrable, then $XY$ is integrable.
$endgroup$
– Will M.
Dec 18 '18 at 1:36












$begingroup$
@WillM. math.stackexchange.com/a/1520480/6179
$endgroup$
– Did
Dec 18 '18 at 7:20




$begingroup$
@WillM. math.stackexchange.com/a/1520480/6179
$endgroup$
– Did
Dec 18 '18 at 7:20










1 Answer
1






active

oldest

votes


















0












$begingroup$

Partial solution.



Since you have a), I give a proof of b). Observe the hypothesis $mathbf{E}(Y mid mathscr{A}) = X$ implies $X$ to be measurable with respect to $mathscr{A}.$ Bearing this in mind,
$$begin{align*}
mathbf{E}((Y-X)^2) &= mathbf{E}big( mathbf{E}((Y-X)^2 mid mathscr{A}) big) \
&= mathbf{E}(mathbf{E}(Y^2 mid mathscr{A}) - 2mathbf{E}(Y mid mathscr{A}) X+ X^2) \
&= mathbf{E}(mathbf{E}(Y^2 mid mathscr{A}) - X^2) = 0.
end{align*}$$



As for c), if $X$ and $Y$ were square integrable, you have $mathbf{E}(XY) = E(X^2) = E(Y^2),$ so b) finishes the proof upon applying it with $mathscr{A} = sigma(X).$ For the general case, I am not sure how to tackle it.






share|cite|improve this answer









$endgroup$













  • $begingroup$
    Hi and thanks for your quick respond! I have a question about the second equal-sign. Why is it $X^2$ and not $mathbb{E}[X|mathcal{A}]$? I think it doesn't make much difference in the result, due to if i write it this way I get zero as well, but maybe I missed something so my way might be faulty (?)
    $endgroup$
    – TNicky
    Dec 17 '18 at 22:46










  • $begingroup$
    You would need $mathbf{E}(X^2 mid mathscr{A}) = X^2$ since $X$ is $mathscr{A}$-measurable.
    $endgroup$
    – Will M.
    Dec 18 '18 at 1:35










  • $begingroup$
    Ok! But I don't see, why $X$ is $mathcal{A}$-measurable. We also need this for the step with the term $mathbb{E}[XY|mathcal{A}]$.
    $endgroup$
    – TNicky
    Dec 18 '18 at 7:52










  • $begingroup$
    If $X = mathbf{E}(Y mid mathscr{A})$ then $X$ is $mathscr{A}$-measurable. If this is not clear, you need to read again the definition of conditional expectation.
    $endgroup$
    – Will M.
    Dec 18 '18 at 7:53










  • $begingroup$
    AAh sure it is clear! Stupid me! Thank you very much!
    $endgroup$
    – TNicky
    Dec 18 '18 at 7:55











Your Answer





StackExchange.ifUsing("editor", function () {
return StackExchange.using("mathjaxEditing", function () {
StackExchange.MarkdownEditor.creationCallbacks.add(function (editor, postfix) {
StackExchange.mathjaxEditing.prepareWmdForMathJax(editor, postfix, [["$", "$"], ["\\(","\\)"]]);
});
});
}, "mathjax-editing");

StackExchange.ready(function() {
var channelOptions = {
tags: "".split(" "),
id: "69"
};
initTagRenderer("".split(" "), "".split(" "), channelOptions);

StackExchange.using("externalEditor", function() {
// Have to fire editor after snippets, if snippets enabled
if (StackExchange.settings.snippets.snippetsEnabled) {
StackExchange.using("snippets", function() {
createEditor();
});
}
else {
createEditor();
}
});

function createEditor() {
StackExchange.prepareEditor({
heartbeatType: 'answer',
autoActivateHeartbeat: false,
convertImagesToLinks: true,
noModals: true,
showLowRepImageUploadWarning: true,
reputationToPostImages: 10,
bindNavPrevention: true,
postfix: "",
imageUploader: {
brandingHtml: "Powered by u003ca class="icon-imgur-white" href="https://imgur.com/"u003eu003c/au003e",
contentPolicyHtml: "User contributions licensed under u003ca href="https://creativecommons.org/licenses/by-sa/3.0/"u003ecc by-sa 3.0 with attribution requiredu003c/au003e u003ca href="https://stackoverflow.com/legal/content-policy"u003e(content policy)u003c/au003e",
allowUrls: true
},
noCode: true, onDemand: true,
discardSelector: ".discard-answer"
,immediatelyShowMarkdownHelp:true
});


}
});














draft saved

draft discarded


















StackExchange.ready(
function () {
StackExchange.openid.initPostLogin('.new-post-login', 'https%3a%2f%2fmath.stackexchange.com%2fquestions%2f3044434%2fmathbbex2-mathbbey2-and-mathbbey-mathcala-x-p-a-s-rig%23new-answer', 'question_page');
}
);

Post as a guest















Required, but never shown

























1 Answer
1






active

oldest

votes








1 Answer
1






active

oldest

votes









active

oldest

votes






active

oldest

votes









0












$begingroup$

Partial solution.



Since you have a), I give a proof of b). Observe the hypothesis $mathbf{E}(Y mid mathscr{A}) = X$ implies $X$ to be measurable with respect to $mathscr{A}.$ Bearing this in mind,
$$begin{align*}
mathbf{E}((Y-X)^2) &= mathbf{E}big( mathbf{E}((Y-X)^2 mid mathscr{A}) big) \
&= mathbf{E}(mathbf{E}(Y^2 mid mathscr{A}) - 2mathbf{E}(Y mid mathscr{A}) X+ X^2) \
&= mathbf{E}(mathbf{E}(Y^2 mid mathscr{A}) - X^2) = 0.
end{align*}$$



As for c), if $X$ and $Y$ were square integrable, you have $mathbf{E}(XY) = E(X^2) = E(Y^2),$ so b) finishes the proof upon applying it with $mathscr{A} = sigma(X).$ For the general case, I am not sure how to tackle it.






share|cite|improve this answer









$endgroup$













  • $begingroup$
    Hi and thanks for your quick respond! I have a question about the second equal-sign. Why is it $X^2$ and not $mathbb{E}[X|mathcal{A}]$? I think it doesn't make much difference in the result, due to if i write it this way I get zero as well, but maybe I missed something so my way might be faulty (?)
    $endgroup$
    – TNicky
    Dec 17 '18 at 22:46










  • $begingroup$
    You would need $mathbf{E}(X^2 mid mathscr{A}) = X^2$ since $X$ is $mathscr{A}$-measurable.
    $endgroup$
    – Will M.
    Dec 18 '18 at 1:35










  • $begingroup$
    Ok! But I don't see, why $X$ is $mathcal{A}$-measurable. We also need this for the step with the term $mathbb{E}[XY|mathcal{A}]$.
    $endgroup$
    – TNicky
    Dec 18 '18 at 7:52










  • $begingroup$
    If $X = mathbf{E}(Y mid mathscr{A})$ then $X$ is $mathscr{A}$-measurable. If this is not clear, you need to read again the definition of conditional expectation.
    $endgroup$
    – Will M.
    Dec 18 '18 at 7:53










  • $begingroup$
    AAh sure it is clear! Stupid me! Thank you very much!
    $endgroup$
    – TNicky
    Dec 18 '18 at 7:55
















0












$begingroup$

Partial solution.



Since you have a), I give a proof of b). Observe the hypothesis $mathbf{E}(Y mid mathscr{A}) = X$ implies $X$ to be measurable with respect to $mathscr{A}.$ Bearing this in mind,
$$begin{align*}
mathbf{E}((Y-X)^2) &= mathbf{E}big( mathbf{E}((Y-X)^2 mid mathscr{A}) big) \
&= mathbf{E}(mathbf{E}(Y^2 mid mathscr{A}) - 2mathbf{E}(Y mid mathscr{A}) X+ X^2) \
&= mathbf{E}(mathbf{E}(Y^2 mid mathscr{A}) - X^2) = 0.
end{align*}$$



As for c), if $X$ and $Y$ were square integrable, you have $mathbf{E}(XY) = E(X^2) = E(Y^2),$ so b) finishes the proof upon applying it with $mathscr{A} = sigma(X).$ For the general case, I am not sure how to tackle it.






share|cite|improve this answer









$endgroup$













  • $begingroup$
    Hi and thanks for your quick respond! I have a question about the second equal-sign. Why is it $X^2$ and not $mathbb{E}[X|mathcal{A}]$? I think it doesn't make much difference in the result, due to if i write it this way I get zero as well, but maybe I missed something so my way might be faulty (?)
    $endgroup$
    – TNicky
    Dec 17 '18 at 22:46










  • $begingroup$
    You would need $mathbf{E}(X^2 mid mathscr{A}) = X^2$ since $X$ is $mathscr{A}$-measurable.
    $endgroup$
    – Will M.
    Dec 18 '18 at 1:35










  • $begingroup$
    Ok! But I don't see, why $X$ is $mathcal{A}$-measurable. We also need this for the step with the term $mathbb{E}[XY|mathcal{A}]$.
    $endgroup$
    – TNicky
    Dec 18 '18 at 7:52










  • $begingroup$
    If $X = mathbf{E}(Y mid mathscr{A})$ then $X$ is $mathscr{A}$-measurable. If this is not clear, you need to read again the definition of conditional expectation.
    $endgroup$
    – Will M.
    Dec 18 '18 at 7:53










  • $begingroup$
    AAh sure it is clear! Stupid me! Thank you very much!
    $endgroup$
    – TNicky
    Dec 18 '18 at 7:55














0












0








0





$begingroup$

Partial solution.



Since you have a), I give a proof of b). Observe the hypothesis $mathbf{E}(Y mid mathscr{A}) = X$ implies $X$ to be measurable with respect to $mathscr{A}.$ Bearing this in mind,
$$begin{align*}
mathbf{E}((Y-X)^2) &= mathbf{E}big( mathbf{E}((Y-X)^2 mid mathscr{A}) big) \
&= mathbf{E}(mathbf{E}(Y^2 mid mathscr{A}) - 2mathbf{E}(Y mid mathscr{A}) X+ X^2) \
&= mathbf{E}(mathbf{E}(Y^2 mid mathscr{A}) - X^2) = 0.
end{align*}$$



As for c), if $X$ and $Y$ were square integrable, you have $mathbf{E}(XY) = E(X^2) = E(Y^2),$ so b) finishes the proof upon applying it with $mathscr{A} = sigma(X).$ For the general case, I am not sure how to tackle it.






share|cite|improve this answer









$endgroup$



Partial solution.



Since you have a), I give a proof of b). Observe the hypothesis $mathbf{E}(Y mid mathscr{A}) = X$ implies $X$ to be measurable with respect to $mathscr{A}.$ Bearing this in mind,
$$begin{align*}
mathbf{E}((Y-X)^2) &= mathbf{E}big( mathbf{E}((Y-X)^2 mid mathscr{A}) big) \
&= mathbf{E}(mathbf{E}(Y^2 mid mathscr{A}) - 2mathbf{E}(Y mid mathscr{A}) X+ X^2) \
&= mathbf{E}(mathbf{E}(Y^2 mid mathscr{A}) - X^2) = 0.
end{align*}$$



As for c), if $X$ and $Y$ were square integrable, you have $mathbf{E}(XY) = E(X^2) = E(Y^2),$ so b) finishes the proof upon applying it with $mathscr{A} = sigma(X).$ For the general case, I am not sure how to tackle it.







share|cite|improve this answer












share|cite|improve this answer



share|cite|improve this answer










answered Dec 17 '18 at 22:11









Will M.Will M.

2,865315




2,865315












  • $begingroup$
    Hi and thanks for your quick respond! I have a question about the second equal-sign. Why is it $X^2$ and not $mathbb{E}[X|mathcal{A}]$? I think it doesn't make much difference in the result, due to if i write it this way I get zero as well, but maybe I missed something so my way might be faulty (?)
    $endgroup$
    – TNicky
    Dec 17 '18 at 22:46










  • $begingroup$
    You would need $mathbf{E}(X^2 mid mathscr{A}) = X^2$ since $X$ is $mathscr{A}$-measurable.
    $endgroup$
    – Will M.
    Dec 18 '18 at 1:35










  • $begingroup$
    Ok! But I don't see, why $X$ is $mathcal{A}$-measurable. We also need this for the step with the term $mathbb{E}[XY|mathcal{A}]$.
    $endgroup$
    – TNicky
    Dec 18 '18 at 7:52










  • $begingroup$
    If $X = mathbf{E}(Y mid mathscr{A})$ then $X$ is $mathscr{A}$-measurable. If this is not clear, you need to read again the definition of conditional expectation.
    $endgroup$
    – Will M.
    Dec 18 '18 at 7:53










  • $begingroup$
    AAh sure it is clear! Stupid me! Thank you very much!
    $endgroup$
    – TNicky
    Dec 18 '18 at 7:55


















  • $begingroup$
    Hi and thanks for your quick respond! I have a question about the second equal-sign. Why is it $X^2$ and not $mathbb{E}[X|mathcal{A}]$? I think it doesn't make much difference in the result, due to if i write it this way I get zero as well, but maybe I missed something so my way might be faulty (?)
    $endgroup$
    – TNicky
    Dec 17 '18 at 22:46










  • $begingroup$
    You would need $mathbf{E}(X^2 mid mathscr{A}) = X^2$ since $X$ is $mathscr{A}$-measurable.
    $endgroup$
    – Will M.
    Dec 18 '18 at 1:35










  • $begingroup$
    Ok! But I don't see, why $X$ is $mathcal{A}$-measurable. We also need this for the step with the term $mathbb{E}[XY|mathcal{A}]$.
    $endgroup$
    – TNicky
    Dec 18 '18 at 7:52










  • $begingroup$
    If $X = mathbf{E}(Y mid mathscr{A})$ then $X$ is $mathscr{A}$-measurable. If this is not clear, you need to read again the definition of conditional expectation.
    $endgroup$
    – Will M.
    Dec 18 '18 at 7:53










  • $begingroup$
    AAh sure it is clear! Stupid me! Thank you very much!
    $endgroup$
    – TNicky
    Dec 18 '18 at 7:55
















$begingroup$
Hi and thanks for your quick respond! I have a question about the second equal-sign. Why is it $X^2$ and not $mathbb{E}[X|mathcal{A}]$? I think it doesn't make much difference in the result, due to if i write it this way I get zero as well, but maybe I missed something so my way might be faulty (?)
$endgroup$
– TNicky
Dec 17 '18 at 22:46




$begingroup$
Hi and thanks for your quick respond! I have a question about the second equal-sign. Why is it $X^2$ and not $mathbb{E}[X|mathcal{A}]$? I think it doesn't make much difference in the result, due to if i write it this way I get zero as well, but maybe I missed something so my way might be faulty (?)
$endgroup$
– TNicky
Dec 17 '18 at 22:46












$begingroup$
You would need $mathbf{E}(X^2 mid mathscr{A}) = X^2$ since $X$ is $mathscr{A}$-measurable.
$endgroup$
– Will M.
Dec 18 '18 at 1:35




$begingroup$
You would need $mathbf{E}(X^2 mid mathscr{A}) = X^2$ since $X$ is $mathscr{A}$-measurable.
$endgroup$
– Will M.
Dec 18 '18 at 1:35












$begingroup$
Ok! But I don't see, why $X$ is $mathcal{A}$-measurable. We also need this for the step with the term $mathbb{E}[XY|mathcal{A}]$.
$endgroup$
– TNicky
Dec 18 '18 at 7:52




$begingroup$
Ok! But I don't see, why $X$ is $mathcal{A}$-measurable. We also need this for the step with the term $mathbb{E}[XY|mathcal{A}]$.
$endgroup$
– TNicky
Dec 18 '18 at 7:52












$begingroup$
If $X = mathbf{E}(Y mid mathscr{A})$ then $X$ is $mathscr{A}$-measurable. If this is not clear, you need to read again the definition of conditional expectation.
$endgroup$
– Will M.
Dec 18 '18 at 7:53




$begingroup$
If $X = mathbf{E}(Y mid mathscr{A})$ then $X$ is $mathscr{A}$-measurable. If this is not clear, you need to read again the definition of conditional expectation.
$endgroup$
– Will M.
Dec 18 '18 at 7:53












$begingroup$
AAh sure it is clear! Stupid me! Thank you very much!
$endgroup$
– TNicky
Dec 18 '18 at 7:55




$begingroup$
AAh sure it is clear! Stupid me! Thank you very much!
$endgroup$
– TNicky
Dec 18 '18 at 7:55


















draft saved

draft discarded




















































Thanks for contributing an answer to Mathematics Stack Exchange!


  • Please be sure to answer the question. Provide details and share your research!

But avoid



  • Asking for help, clarification, or responding to other answers.

  • Making statements based on opinion; back them up with references or personal experience.


Use MathJax to format equations. MathJax reference.


To learn more, see our tips on writing great answers.




draft saved


draft discarded














StackExchange.ready(
function () {
StackExchange.openid.initPostLogin('.new-post-login', 'https%3a%2f%2fmath.stackexchange.com%2fquestions%2f3044434%2fmathbbex2-mathbbey2-and-mathbbey-mathcala-x-p-a-s-rig%23new-answer', 'question_page');
}
);

Post as a guest















Required, but never shown





















































Required, but never shown














Required, but never shown












Required, but never shown







Required, but never shown

































Required, but never shown














Required, but never shown












Required, but never shown







Required, but never shown







Popular posts from this blog

Plaza Victoria

In PowerPoint, is there a keyboard shortcut for bulleted / numbered list?

How to put 3 figures in Latex with 2 figures side by side and 1 below these side by side images but in...